Question

Which of the following statements concerning the variance are correct? (note: there can be more than...

Which of the following statements concerning the variance are correct? (note: there can be more than one correct answer)

Group of answer choices

a.The larger the variance, the greater the total risk of the investment.

b.If a stock portfolio is well diversified, then the portfolio variance may be less than the variance of the least risky stock in the portfolio.

c.The larger the variance, the smaller the standard deviation.

d.The larger the variance, the more the actual returns tend to differ from the average return.

e.The variance is the best measure of risk for an asset held in isolation.

Homework Answers

Answer #1

The correct statements are:

a.The larger the variance, the greater the total risk of the investment.

b.If a stock portfolio is well diversified, then the portfolio variance may be less than the variance of the least risky stock in the portfolio.

d.The larger the variance, the more the actual returns tend to differ from the average return.

and

e.The variance is the best measure of risk for an asset held in isolation.

Variance is a meaure of risk anf higher the variance, higher will be the Standard Deviation

Variation represents difference in actual return than average return

Diversification reduces risk

Hence, all statements except c are correct

Know the answer?
Your Answer:

Post as a guest

Your Name:

What's your source?

Earn Coins

Coins can be redeemed for fabulous gifts.

Not the answer you're looking for?
Ask your own homework help question
Similar Questions
Which of the following statements is CORRECT? Select one: a. The beta of a portfolio of...
Which of the following statements is CORRECT? Select one: a. The beta of a portfolio of stocks is always smaller than the betas of any of the individual stocks. b. The beta of a portfolio of stocks is always larger than the betas of any of the individual stocks. c. It is theoretically possible for a stock to have a beta of 1.0. If a stock did have a beta of 1.0, then, at least in theory, its required rate...
choose more than one correct answer If the F ratio in an analysis of variance is...
choose more than one correct answer If the F ratio in an analysis of variance is much greater than 1, then: Group of answer choices The within groups estimate of the population variance is much smaller than the between-groups estimate The within groups estimate of the population variance is much larger than the between-groups estimate You fail to reject the null hypothesis You reject the null hypothesis
2. Which of the following statements concerning beta is correct? a. A stock with a beta...
2. Which of the following statements concerning beta is correct? a. A stock with a beta of 0 is expected to provide a rate of return equal to the market portfolio b. A stock with a beta equal to 1 has no risk c. Stocks with negative betas have the least amount of risk FALSE d. A stock with a beta greater than 1 is expected to be more volatile than the market portfolio
Which of the following statements is correct?(x)Adding stocks to your portfolio can reduce firm-specific risk, but...
Which of the following statements is correct?(x)Adding stocks to your portfolio can reduce firm-specific risk, but you will not eliminate market risk.(y)A low standard deviation means that the investment is less likely to achieve a much higher return than its average, but a low standard deviation indicates that the investment is less risky.(z)Expected returns may differ from actual returns because of an unforeseen economic expansion. A.(x), (y) and (z)B.(x) and (y) onlyC.(x) and (z) onlyD.(y) and (z) onlyE.(x) only A...
Which two of the following five statements are correct? Select two alternatives: While the variance and...
Which two of the following five statements are correct? Select two alternatives: While the variance and the standard deviation both measure the variability of the returns, the variance is easier to interpret because it is in the same units as the returns themselves. The 95% confidence interval for the expected return is defined as the Historical Average Return plus or minus three standard errors. We can use a security's historical average return to estimate its actual expected return. The geometric...
A stock's beta is more relevant as a measure of risk to an investor who holds...
A stock's beta is more relevant as a measure of risk to an investor who holds only one stock than to an investor who holds a well-diversified portfolio. Group of answer choices True False
Stock A's beta is 1.5 and Stock B's beta is 0.5. Which of the following statements...
Stock A's beta is 1.5 and Stock B's beta is 0.5. Which of the following statements must be true, assuming the CAPM is correct. a. Stock A would be a more desirable addition to a portfolio then Stock B. b. Stock B would be a more desirable addition to a portfolio than A. c. When held in isolation, Stock A has more risk than Stock B. d. In equilibrium, the expected return on Stock B will be greater than that...
Stock A's beta is 1.5 and Stock B's beta is 0.5. Which of the following statements...
Stock A's beta is 1.5 and Stock B's beta is 0.5. Which of the following statements must be true, assuming the CAPM is correct. a. Stock A would be a more desirable addition to a portfolio then Stock B. b. In equilibrium, the expected return on Stock B will be greater than that on Stock A. c. When held in isolation, Stock A has more risk than Stock B. d. Stock B would be a more desirable addition to a...
QUESTION 1 Part A: Which of the following statements is CORRECT? a. An investor can eliminate...
QUESTION 1 Part A: Which of the following statements is CORRECT? a. An investor can eliminate virtually all stand-alone risk if he or she holds a very large and well diversified portfolio of stocks. b. The higher the correlation between the stocks in a portfolio, the lower the risk inherent in the portfolio. c. Once a portfolio has about 40 stocks, adding additional stocks will not reduce its risk by even a small amount. d. An investor can eliminate virtually...
Stock X has a 9.5% expected return, a beta coefficient of 0.8, and a 30% standard...
Stock X has a 9.5% expected return, a beta coefficient of 0.8, and a 30% standard deviation of expected returns. Stock Y has a 12.0% expected return, a beta coefficient of 1.1, and a 25.0% standard deviation. The risk-free rate is 6%, and the market risk premium is 5%. Calculate each stock's coefficient of variation. Round your answers to two decimal places. Do not round intermediate calculations. CVx = CVy = Which stock is riskier for a diversified investor? For...
ADVERTISEMENT
Need Online Homework Help?

Get Answers For Free
Most questions answered within 1 hours.

Ask a Question
ADVERTISEMENT